How to show that $H$ is normal in $G$?how to show that $P$ is normal complement in $H$.Prove H is a normal subgroup of G.Please help me with proving a subgroup is normalShow that $D$ is a normal subgroup.Showing that every subgroup of an abelian group is normalFor all $gin G$, is it true that $gHg^-1 subseteq H $ if H is a normal subgroup of G.An exercise about normal subgroupsOrder of normal subgroupHow to show that a subgroup is normal.$PQ$ is normal implies $P$ and $Q$ are also normal

Why is the design of haulage companies so “special”?

Do Phineas and Ferb ever actually get busted in real time?

When blogging recipes, how can I support both readers who want the narrative/journey and ones who want the printer-friendly recipe?

How to report a triplet of septets in NMR tabulation?

Why is an old chain unsafe?

Why don't electromagnetic waves interact with each other?

Are tax years 2016 & 2017 back taxes deductible for tax year 2018?

Why is this code 6.5x slower with optimizations enabled?

Banach space and Hilbert space topology

What makes Graph invariants so useful/important?

Copenhagen passport control - US citizen

How to add power-LED to my small amplifier?

What do you call something that goes against the spirit of the law, but is legal when interpreting the law to the letter?

What would the Romans have called "sorcery"?

How do we improve the relationship with a client software team that performs poorly and is becoming less collaborative?

What is the command to reset a PC without deleting any files

How is this relation reflexive?

GPS Rollover on Android Smartphones

What defenses are there against being summoned by the Gate spell?

Prevent a directory in /tmp from being deleted

Set-theoretical foundations of Mathematics with only bounded quantifiers

What typically incentivizes a professor to change jobs to a lower ranking university?

Is Social Media Science Fiction?

Why are only specific transaction types accepted into the mempool?



How to show that $H$ is normal in $G$?


how to show that $P$ is normal complement in $H$.Prove H is a normal subgroup of G.Please help me with proving a subgroup is normalShow that $D$ is a normal subgroup.Showing that every subgroup of an abelian group is normalFor all $gin G$, is it true that $gHg^-1 subseteq H $ if H is a normal subgroup of G.An exercise about normal subgroupsOrder of normal subgroupHow to show that a subgroup is normal.$PQ$ is normal implies $P$ and $Q$ are also normal













2












$begingroup$


Let $G$ be a group of order $pq$, where $p$ and $q$ are primes and $p>q$. Ler $ain G$ be of order $p$ and $H=big<a: a^p=1big>.$ Then $H$ is normal in $G$.
I know that $H$ will be normal subgroup of G if $gH=Hg$ or $H=gHg^-1$.
I tried as:



Let $gin G$ and $hin H$ then $h$ can be written in some power of $a$ but I don't, how to write $g$ and proceed to show that $H=ghg^-1$.










share|cite|improve this question











$endgroup$







  • 1




    $begingroup$
    Are you familiar with Sylow's theorem?
    $endgroup$
    – Thomas Shelby
    Mar 22 at 8:56










  • $begingroup$
    It suffices to show $g^-1Hgsubseteq H$ for all $gin G$.
    $endgroup$
    – Shaun
    Mar 22 at 9:00















2












$begingroup$


Let $G$ be a group of order $pq$, where $p$ and $q$ are primes and $p>q$. Ler $ain G$ be of order $p$ and $H=big<a: a^p=1big>.$ Then $H$ is normal in $G$.
I know that $H$ will be normal subgroup of G if $gH=Hg$ or $H=gHg^-1$.
I tried as:



Let $gin G$ and $hin H$ then $h$ can be written in some power of $a$ but I don't, how to write $g$ and proceed to show that $H=ghg^-1$.










share|cite|improve this question











$endgroup$







  • 1




    $begingroup$
    Are you familiar with Sylow's theorem?
    $endgroup$
    – Thomas Shelby
    Mar 22 at 8:56










  • $begingroup$
    It suffices to show $g^-1Hgsubseteq H$ for all $gin G$.
    $endgroup$
    – Shaun
    Mar 22 at 9:00













2












2








2





$begingroup$


Let $G$ be a group of order $pq$, where $p$ and $q$ are primes and $p>q$. Ler $ain G$ be of order $p$ and $H=big<a: a^p=1big>.$ Then $H$ is normal in $G$.
I know that $H$ will be normal subgroup of G if $gH=Hg$ or $H=gHg^-1$.
I tried as:



Let $gin G$ and $hin H$ then $h$ can be written in some power of $a$ but I don't, how to write $g$ and proceed to show that $H=ghg^-1$.










share|cite|improve this question











$endgroup$




Let $G$ be a group of order $pq$, where $p$ and $q$ are primes and $p>q$. Ler $ain G$ be of order $p$ and $H=big<a: a^p=1big>.$ Then $H$ is normal in $G$.
I know that $H$ will be normal subgroup of G if $gH=Hg$ or $H=gHg^-1$.
I tried as:



Let $gin G$ and $hin H$ then $h$ can be written in some power of $a$ but I don't, how to write $g$ and proceed to show that $H=ghg^-1$.







group-theory normal-subgroups






share|cite|improve this question















share|cite|improve this question













share|cite|improve this question




share|cite|improve this question








edited Mar 22 at 9:03









Shaun

10.4k113686




10.4k113686










asked Mar 22 at 8:51









Noor AslamNoor Aslam

16012




16012







  • 1




    $begingroup$
    Are you familiar with Sylow's theorem?
    $endgroup$
    – Thomas Shelby
    Mar 22 at 8:56










  • $begingroup$
    It suffices to show $g^-1Hgsubseteq H$ for all $gin G$.
    $endgroup$
    – Shaun
    Mar 22 at 9:00












  • 1




    $begingroup$
    Are you familiar with Sylow's theorem?
    $endgroup$
    – Thomas Shelby
    Mar 22 at 8:56










  • $begingroup$
    It suffices to show $g^-1Hgsubseteq H$ for all $gin G$.
    $endgroup$
    – Shaun
    Mar 22 at 9:00







1




1




$begingroup$
Are you familiar with Sylow's theorem?
$endgroup$
– Thomas Shelby
Mar 22 at 8:56




$begingroup$
Are you familiar with Sylow's theorem?
$endgroup$
– Thomas Shelby
Mar 22 at 8:56












$begingroup$
It suffices to show $g^-1Hgsubseteq H$ for all $gin G$.
$endgroup$
– Shaun
Mar 22 at 9:00




$begingroup$
It suffices to show $g^-1Hgsubseteq H$ for all $gin G$.
$endgroup$
– Shaun
Mar 22 at 9:00










2 Answers
2






active

oldest

votes


















0












$begingroup$

The proof above very nearly works, so let's finish it up.



Lemma: If $x^p=1, text then forall g in G (g^-1xg)^p=1,$



Proof: Since consecutive pairs $g^-1g=1$ cancel each other, we have $(g^-1xg)^p=g^-1x^pg=g^-1g=1.$



Now assume $a in H$. Then $exists x_1, x_2, ldots x_n in H text such that x_k^p=1 text and a=x_1x_2cdots x_n$. Then $$g^-1ag= prod_k=1^n g^-1x_kg.$$



By the Lemma, each $g^-1x_kg$ is a generator of $H$ because it has order $p$, so $g^-1ag in H$ and since $a in H, g in G$ were arbitrary, that shows $forall g in G~g^-1Hg subseteq H$ so $H triangleleft G$ and we are done.






share|cite|improve this answer









$endgroup$




















    2












    $begingroup$

    Let's take any $a$ from $H$. Let's take some other element $g$ from $G$ and examine what is $x = gag^-1$. But



    $$x^p = g a g^-1 g a g^-1...g a g^-1 = g a^p g^-1 = g e g^-1 = e$$



    So, $x$ also belongs to $H$.



    Conjugating by any element of the group $G$ leaves any element of $H$ inside $H$. That's actually a definition of normal subgroup. So, $H$ is normal in $G$.



    UPDATE



    As Tobias Kildetoft pointed out, this is not a proof yet. It is only a proof that conjugating leaves the set of all elements $S = a: a^p=1$ in place. The subgroup generated by these elements is a different thing!



    Let's take some element $h$ of $H$. It is generated by elements of $S$:
    $$ h = s_1 s_2... s_n $$



    It's conjugated element:
    $$ ghg^-1 = gs_1 s_2... s_ng^-1 = gs_1g^-1 gs_2g^-1... gs_ng^-1 $$



    Is generated by $gs_ig^-1$. So, it is generated by by the elements of the same set $S$. So, it also belongs to $H$, which means $H$ is normal in $G$.






    share|cite|improve this answer











    $endgroup$








    • 1




      $begingroup$
      The idea is correct, but there is a small detail missing: $H$ does not necessarily just consist of elements of order dividing $p$, but is generated by those.
      $endgroup$
      – Tobias Kildetoft
      Mar 22 at 9:08






    • 1




      $begingroup$
      @ThomasShelby No, it is the subgroup generated by all elements of order $p$.
      $endgroup$
      – Tobias Kildetoft
      Mar 22 at 9:12










    • $begingroup$
      @ThomasShelby I guess clarification is needed from the author. $H=big<a: a^p=1big>.$ - doesn't it read as "all $a$ such that ..."?
      $endgroup$
      – lesnik
      Mar 22 at 9:12






    • 1




      $begingroup$
      @ThomasShelby Until you have shown that the subgroup in question is in fact of order $p$, you can't use Sylow. And the conclusion that it is of order $p$ is the same as there being just one Sylow $p$-subgroup (the subgroup in question is always normal, regardless of the group, it just happens to be of a nice form in this case).
      $endgroup$
      – Tobias Kildetoft
      Mar 22 at 9:19







    • 1




      $begingroup$
      @ThomasShelby Sure, but that is not the subgroup we care about here (except that it turns out to be the same due to the order of the group).
      $endgroup$
      – Tobias Kildetoft
      Mar 22 at 9:30











    Your Answer





    StackExchange.ifUsing("editor", function ()
    return StackExchange.using("mathjaxEditing", function ()
    StackExchange.MarkdownEditor.creationCallbacks.add(function (editor, postfix)
    StackExchange.mathjaxEditing.prepareWmdForMathJax(editor, postfix, [["$", "$"], ["\\(","\\)"]]);
    );
    );
    , "mathjax-editing");

    StackExchange.ready(function()
    var channelOptions =
    tags: "".split(" "),
    id: "69"
    ;
    initTagRenderer("".split(" "), "".split(" "), channelOptions);

    StackExchange.using("externalEditor", function()
    // Have to fire editor after snippets, if snippets enabled
    if (StackExchange.settings.snippets.snippetsEnabled)
    StackExchange.using("snippets", function()
    createEditor();
    );

    else
    createEditor();

    );

    function createEditor()
    StackExchange.prepareEditor(
    heartbeatType: 'answer',
    autoActivateHeartbeat: false,
    convertImagesToLinks: true,
    noModals: true,
    showLowRepImageUploadWarning: true,
    reputationToPostImages: 10,
    bindNavPrevention: true,
    postfix: "",
    imageUploader:
    brandingHtml: "Powered by u003ca class="icon-imgur-white" href="https://imgur.com/"u003eu003c/au003e",
    contentPolicyHtml: "User contributions licensed under u003ca href="https://creativecommons.org/licenses/by-sa/3.0/"u003ecc by-sa 3.0 with attribution requiredu003c/au003e u003ca href="https://stackoverflow.com/legal/content-policy"u003e(content policy)u003c/au003e",
    allowUrls: true
    ,
    noCode: true, onDemand: true,
    discardSelector: ".discard-answer"
    ,immediatelyShowMarkdownHelp:true
    );



    );













    draft saved

    draft discarded


















    StackExchange.ready(
    function ()
    StackExchange.openid.initPostLogin('.new-post-login', 'https%3a%2f%2fmath.stackexchange.com%2fquestions%2f3157909%2fhow-to-show-that-h-is-normal-in-g%23new-answer', 'question_page');

    );

    Post as a guest















    Required, but never shown

























    2 Answers
    2






    active

    oldest

    votes








    2 Answers
    2






    active

    oldest

    votes









    active

    oldest

    votes






    active

    oldest

    votes









    0












    $begingroup$

    The proof above very nearly works, so let's finish it up.



    Lemma: If $x^p=1, text then forall g in G (g^-1xg)^p=1,$



    Proof: Since consecutive pairs $g^-1g=1$ cancel each other, we have $(g^-1xg)^p=g^-1x^pg=g^-1g=1.$



    Now assume $a in H$. Then $exists x_1, x_2, ldots x_n in H text such that x_k^p=1 text and a=x_1x_2cdots x_n$. Then $$g^-1ag= prod_k=1^n g^-1x_kg.$$



    By the Lemma, each $g^-1x_kg$ is a generator of $H$ because it has order $p$, so $g^-1ag in H$ and since $a in H, g in G$ were arbitrary, that shows $forall g in G~g^-1Hg subseteq H$ so $H triangleleft G$ and we are done.






    share|cite|improve this answer









    $endgroup$

















      0












      $begingroup$

      The proof above very nearly works, so let's finish it up.



      Lemma: If $x^p=1, text then forall g in G (g^-1xg)^p=1,$



      Proof: Since consecutive pairs $g^-1g=1$ cancel each other, we have $(g^-1xg)^p=g^-1x^pg=g^-1g=1.$



      Now assume $a in H$. Then $exists x_1, x_2, ldots x_n in H text such that x_k^p=1 text and a=x_1x_2cdots x_n$. Then $$g^-1ag= prod_k=1^n g^-1x_kg.$$



      By the Lemma, each $g^-1x_kg$ is a generator of $H$ because it has order $p$, so $g^-1ag in H$ and since $a in H, g in G$ were arbitrary, that shows $forall g in G~g^-1Hg subseteq H$ so $H triangleleft G$ and we are done.






      share|cite|improve this answer









      $endgroup$















        0












        0








        0





        $begingroup$

        The proof above very nearly works, so let's finish it up.



        Lemma: If $x^p=1, text then forall g in G (g^-1xg)^p=1,$



        Proof: Since consecutive pairs $g^-1g=1$ cancel each other, we have $(g^-1xg)^p=g^-1x^pg=g^-1g=1.$



        Now assume $a in H$. Then $exists x_1, x_2, ldots x_n in H text such that x_k^p=1 text and a=x_1x_2cdots x_n$. Then $$g^-1ag= prod_k=1^n g^-1x_kg.$$



        By the Lemma, each $g^-1x_kg$ is a generator of $H$ because it has order $p$, so $g^-1ag in H$ and since $a in H, g in G$ were arbitrary, that shows $forall g in G~g^-1Hg subseteq H$ so $H triangleleft G$ and we are done.






        share|cite|improve this answer









        $endgroup$



        The proof above very nearly works, so let's finish it up.



        Lemma: If $x^p=1, text then forall g in G (g^-1xg)^p=1,$



        Proof: Since consecutive pairs $g^-1g=1$ cancel each other, we have $(g^-1xg)^p=g^-1x^pg=g^-1g=1.$



        Now assume $a in H$. Then $exists x_1, x_2, ldots x_n in H text such that x_k^p=1 text and a=x_1x_2cdots x_n$. Then $$g^-1ag= prod_k=1^n g^-1x_kg.$$



        By the Lemma, each $g^-1x_kg$ is a generator of $H$ because it has order $p$, so $g^-1ag in H$ and since $a in H, g in G$ were arbitrary, that shows $forall g in G~g^-1Hg subseteq H$ so $H triangleleft G$ and we are done.







        share|cite|improve this answer












        share|cite|improve this answer



        share|cite|improve this answer










        answered Mar 22 at 9:41









        Robert ShoreRobert Shore

        3,611324




        3,611324





















            2












            $begingroup$

            Let's take any $a$ from $H$. Let's take some other element $g$ from $G$ and examine what is $x = gag^-1$. But



            $$x^p = g a g^-1 g a g^-1...g a g^-1 = g a^p g^-1 = g e g^-1 = e$$



            So, $x$ also belongs to $H$.



            Conjugating by any element of the group $G$ leaves any element of $H$ inside $H$. That's actually a definition of normal subgroup. So, $H$ is normal in $G$.



            UPDATE



            As Tobias Kildetoft pointed out, this is not a proof yet. It is only a proof that conjugating leaves the set of all elements $S = a: a^p=1$ in place. The subgroup generated by these elements is a different thing!



            Let's take some element $h$ of $H$. It is generated by elements of $S$:
            $$ h = s_1 s_2... s_n $$



            It's conjugated element:
            $$ ghg^-1 = gs_1 s_2... s_ng^-1 = gs_1g^-1 gs_2g^-1... gs_ng^-1 $$



            Is generated by $gs_ig^-1$. So, it is generated by by the elements of the same set $S$. So, it also belongs to $H$, which means $H$ is normal in $G$.






            share|cite|improve this answer











            $endgroup$








            • 1




              $begingroup$
              The idea is correct, but there is a small detail missing: $H$ does not necessarily just consist of elements of order dividing $p$, but is generated by those.
              $endgroup$
              – Tobias Kildetoft
              Mar 22 at 9:08






            • 1




              $begingroup$
              @ThomasShelby No, it is the subgroup generated by all elements of order $p$.
              $endgroup$
              – Tobias Kildetoft
              Mar 22 at 9:12










            • $begingroup$
              @ThomasShelby I guess clarification is needed from the author. $H=big<a: a^p=1big>.$ - doesn't it read as "all $a$ such that ..."?
              $endgroup$
              – lesnik
              Mar 22 at 9:12






            • 1




              $begingroup$
              @ThomasShelby Until you have shown that the subgroup in question is in fact of order $p$, you can't use Sylow. And the conclusion that it is of order $p$ is the same as there being just one Sylow $p$-subgroup (the subgroup in question is always normal, regardless of the group, it just happens to be of a nice form in this case).
              $endgroup$
              – Tobias Kildetoft
              Mar 22 at 9:19







            • 1




              $begingroup$
              @ThomasShelby Sure, but that is not the subgroup we care about here (except that it turns out to be the same due to the order of the group).
              $endgroup$
              – Tobias Kildetoft
              Mar 22 at 9:30















            2












            $begingroup$

            Let's take any $a$ from $H$. Let's take some other element $g$ from $G$ and examine what is $x = gag^-1$. But



            $$x^p = g a g^-1 g a g^-1...g a g^-1 = g a^p g^-1 = g e g^-1 = e$$



            So, $x$ also belongs to $H$.



            Conjugating by any element of the group $G$ leaves any element of $H$ inside $H$. That's actually a definition of normal subgroup. So, $H$ is normal in $G$.



            UPDATE



            As Tobias Kildetoft pointed out, this is not a proof yet. It is only a proof that conjugating leaves the set of all elements $S = a: a^p=1$ in place. The subgroup generated by these elements is a different thing!



            Let's take some element $h$ of $H$. It is generated by elements of $S$:
            $$ h = s_1 s_2... s_n $$



            It's conjugated element:
            $$ ghg^-1 = gs_1 s_2... s_ng^-1 = gs_1g^-1 gs_2g^-1... gs_ng^-1 $$



            Is generated by $gs_ig^-1$. So, it is generated by by the elements of the same set $S$. So, it also belongs to $H$, which means $H$ is normal in $G$.






            share|cite|improve this answer











            $endgroup$








            • 1




              $begingroup$
              The idea is correct, but there is a small detail missing: $H$ does not necessarily just consist of elements of order dividing $p$, but is generated by those.
              $endgroup$
              – Tobias Kildetoft
              Mar 22 at 9:08






            • 1




              $begingroup$
              @ThomasShelby No, it is the subgroup generated by all elements of order $p$.
              $endgroup$
              – Tobias Kildetoft
              Mar 22 at 9:12










            • $begingroup$
              @ThomasShelby I guess clarification is needed from the author. $H=big<a: a^p=1big>.$ - doesn't it read as "all $a$ such that ..."?
              $endgroup$
              – lesnik
              Mar 22 at 9:12






            • 1




              $begingroup$
              @ThomasShelby Until you have shown that the subgroup in question is in fact of order $p$, you can't use Sylow. And the conclusion that it is of order $p$ is the same as there being just one Sylow $p$-subgroup (the subgroup in question is always normal, regardless of the group, it just happens to be of a nice form in this case).
              $endgroup$
              – Tobias Kildetoft
              Mar 22 at 9:19







            • 1




              $begingroup$
              @ThomasShelby Sure, but that is not the subgroup we care about here (except that it turns out to be the same due to the order of the group).
              $endgroup$
              – Tobias Kildetoft
              Mar 22 at 9:30













            2












            2








            2





            $begingroup$

            Let's take any $a$ from $H$. Let's take some other element $g$ from $G$ and examine what is $x = gag^-1$. But



            $$x^p = g a g^-1 g a g^-1...g a g^-1 = g a^p g^-1 = g e g^-1 = e$$



            So, $x$ also belongs to $H$.



            Conjugating by any element of the group $G$ leaves any element of $H$ inside $H$. That's actually a definition of normal subgroup. So, $H$ is normal in $G$.



            UPDATE



            As Tobias Kildetoft pointed out, this is not a proof yet. It is only a proof that conjugating leaves the set of all elements $S = a: a^p=1$ in place. The subgroup generated by these elements is a different thing!



            Let's take some element $h$ of $H$. It is generated by elements of $S$:
            $$ h = s_1 s_2... s_n $$



            It's conjugated element:
            $$ ghg^-1 = gs_1 s_2... s_ng^-1 = gs_1g^-1 gs_2g^-1... gs_ng^-1 $$



            Is generated by $gs_ig^-1$. So, it is generated by by the elements of the same set $S$. So, it also belongs to $H$, which means $H$ is normal in $G$.






            share|cite|improve this answer











            $endgroup$



            Let's take any $a$ from $H$. Let's take some other element $g$ from $G$ and examine what is $x = gag^-1$. But



            $$x^p = g a g^-1 g a g^-1...g a g^-1 = g a^p g^-1 = g e g^-1 = e$$



            So, $x$ also belongs to $H$.



            Conjugating by any element of the group $G$ leaves any element of $H$ inside $H$. That's actually a definition of normal subgroup. So, $H$ is normal in $G$.



            UPDATE



            As Tobias Kildetoft pointed out, this is not a proof yet. It is only a proof that conjugating leaves the set of all elements $S = a: a^p=1$ in place. The subgroup generated by these elements is a different thing!



            Let's take some element $h$ of $H$. It is generated by elements of $S$:
            $$ h = s_1 s_2... s_n $$



            It's conjugated element:
            $$ ghg^-1 = gs_1 s_2... s_ng^-1 = gs_1g^-1 gs_2g^-1... gs_ng^-1 $$



            Is generated by $gs_ig^-1$. So, it is generated by by the elements of the same set $S$. So, it also belongs to $H$, which means $H$ is normal in $G$.







            share|cite|improve this answer














            share|cite|improve this answer



            share|cite|improve this answer








            edited Mar 22 at 9:54

























            answered Mar 22 at 9:01









            lesniklesnik

            1,495712




            1,495712







            • 1




              $begingroup$
              The idea is correct, but there is a small detail missing: $H$ does not necessarily just consist of elements of order dividing $p$, but is generated by those.
              $endgroup$
              – Tobias Kildetoft
              Mar 22 at 9:08






            • 1




              $begingroup$
              @ThomasShelby No, it is the subgroup generated by all elements of order $p$.
              $endgroup$
              – Tobias Kildetoft
              Mar 22 at 9:12










            • $begingroup$
              @ThomasShelby I guess clarification is needed from the author. $H=big<a: a^p=1big>.$ - doesn't it read as "all $a$ such that ..."?
              $endgroup$
              – lesnik
              Mar 22 at 9:12






            • 1




              $begingroup$
              @ThomasShelby Until you have shown that the subgroup in question is in fact of order $p$, you can't use Sylow. And the conclusion that it is of order $p$ is the same as there being just one Sylow $p$-subgroup (the subgroup in question is always normal, regardless of the group, it just happens to be of a nice form in this case).
              $endgroup$
              – Tobias Kildetoft
              Mar 22 at 9:19







            • 1




              $begingroup$
              @ThomasShelby Sure, but that is not the subgroup we care about here (except that it turns out to be the same due to the order of the group).
              $endgroup$
              – Tobias Kildetoft
              Mar 22 at 9:30












            • 1




              $begingroup$
              The idea is correct, but there is a small detail missing: $H$ does not necessarily just consist of elements of order dividing $p$, but is generated by those.
              $endgroup$
              – Tobias Kildetoft
              Mar 22 at 9:08






            • 1




              $begingroup$
              @ThomasShelby No, it is the subgroup generated by all elements of order $p$.
              $endgroup$
              – Tobias Kildetoft
              Mar 22 at 9:12










            • $begingroup$
              @ThomasShelby I guess clarification is needed from the author. $H=big<a: a^p=1big>.$ - doesn't it read as "all $a$ such that ..."?
              $endgroup$
              – lesnik
              Mar 22 at 9:12






            • 1




              $begingroup$
              @ThomasShelby Until you have shown that the subgroup in question is in fact of order $p$, you can't use Sylow. And the conclusion that it is of order $p$ is the same as there being just one Sylow $p$-subgroup (the subgroup in question is always normal, regardless of the group, it just happens to be of a nice form in this case).
              $endgroup$
              – Tobias Kildetoft
              Mar 22 at 9:19







            • 1




              $begingroup$
              @ThomasShelby Sure, but that is not the subgroup we care about here (except that it turns out to be the same due to the order of the group).
              $endgroup$
              – Tobias Kildetoft
              Mar 22 at 9:30







            1




            1




            $begingroup$
            The idea is correct, but there is a small detail missing: $H$ does not necessarily just consist of elements of order dividing $p$, but is generated by those.
            $endgroup$
            – Tobias Kildetoft
            Mar 22 at 9:08




            $begingroup$
            The idea is correct, but there is a small detail missing: $H$ does not necessarily just consist of elements of order dividing $p$, but is generated by those.
            $endgroup$
            – Tobias Kildetoft
            Mar 22 at 9:08




            1




            1




            $begingroup$
            @ThomasShelby No, it is the subgroup generated by all elements of order $p$.
            $endgroup$
            – Tobias Kildetoft
            Mar 22 at 9:12




            $begingroup$
            @ThomasShelby No, it is the subgroup generated by all elements of order $p$.
            $endgroup$
            – Tobias Kildetoft
            Mar 22 at 9:12












            $begingroup$
            @ThomasShelby I guess clarification is needed from the author. $H=big<a: a^p=1big>.$ - doesn't it read as "all $a$ such that ..."?
            $endgroup$
            – lesnik
            Mar 22 at 9:12




            $begingroup$
            @ThomasShelby I guess clarification is needed from the author. $H=big<a: a^p=1big>.$ - doesn't it read as "all $a$ such that ..."?
            $endgroup$
            – lesnik
            Mar 22 at 9:12




            1




            1




            $begingroup$
            @ThomasShelby Until you have shown that the subgroup in question is in fact of order $p$, you can't use Sylow. And the conclusion that it is of order $p$ is the same as there being just one Sylow $p$-subgroup (the subgroup in question is always normal, regardless of the group, it just happens to be of a nice form in this case).
            $endgroup$
            – Tobias Kildetoft
            Mar 22 at 9:19





            $begingroup$
            @ThomasShelby Until you have shown that the subgroup in question is in fact of order $p$, you can't use Sylow. And the conclusion that it is of order $p$ is the same as there being just one Sylow $p$-subgroup (the subgroup in question is always normal, regardless of the group, it just happens to be of a nice form in this case).
            $endgroup$
            – Tobias Kildetoft
            Mar 22 at 9:19





            1




            1




            $begingroup$
            @ThomasShelby Sure, but that is not the subgroup we care about here (except that it turns out to be the same due to the order of the group).
            $endgroup$
            – Tobias Kildetoft
            Mar 22 at 9:30




            $begingroup$
            @ThomasShelby Sure, but that is not the subgroup we care about here (except that it turns out to be the same due to the order of the group).
            $endgroup$
            – Tobias Kildetoft
            Mar 22 at 9:30

















            draft saved

            draft discarded
















































            Thanks for contributing an answer to Mathematics Stack Exchange!


            • Please be sure to answer the question. Provide details and share your research!

            But avoid


            • Asking for help, clarification, or responding to other answers.

            • Making statements based on opinion; back them up with references or personal experience.

            Use MathJax to format equations. MathJax reference.


            To learn more, see our tips on writing great answers.




            draft saved


            draft discarded














            StackExchange.ready(
            function ()
            StackExchange.openid.initPostLogin('.new-post-login', 'https%3a%2f%2fmath.stackexchange.com%2fquestions%2f3157909%2fhow-to-show-that-h-is-normal-in-g%23new-answer', 'question_page');

            );

            Post as a guest















            Required, but never shown





















































            Required, but never shown














            Required, but never shown












            Required, but never shown







            Required, but never shown

































            Required, but never shown














            Required, but never shown












            Required, but never shown







            Required, but never shown







            Popular posts from this blog

            How should I support this large drywall patch? Planned maintenance scheduled April 23, 2019 at 00:00UTC (8:00pm US/Eastern) Announcing the arrival of Valued Associate #679: Cesar Manara Unicorn Meta Zoo #1: Why another podcast?How do I cover large gaps in drywall?How do I keep drywall around a patch from crumbling?Can I glue a second layer of drywall?How to patch long strip on drywall?Large drywall patch: how to avoid bulging seams?Drywall Mesh Patch vs. Bulge? To remove or not to remove?How to fix this drywall job?Prep drywall before backsplashWhat's the best way to fix this horrible drywall patch job?Drywall patching using 3M Patch Plus Primer

            Lowndes Grove History Architecture References Navigation menu32°48′6″N 79°57′58″W / 32.80167°N 79.96611°W / 32.80167; -79.9661132°48′6″N 79°57′58″W / 32.80167°N 79.96611°W / 32.80167; -79.9661178002500"National Register Information System"Historic houses of South Carolina"Lowndes Grove""+32° 48' 6.00", −79° 57' 58.00""Lowndes Grove, Charleston County (260 St. Margaret St., Charleston)""Lowndes Grove"The Charleston ExpositionIt Happened in South Carolina"Lowndes Grove (House), Saint Margaret Street & Sixth Avenue, Charleston, Charleston County, SC(Photographs)"Plantations of the Carolina Low Countrye

            random experiment with two different functions on unit interval Announcing the arrival of Valued Associate #679: Cesar Manara Planned maintenance scheduled April 23, 2019 at 00:00UTC (8:00pm US/Eastern)Random variable and probability space notionsRandom Walk with EdgesFinding functions where the increase over a random interval is Poisson distributedNumber of days until dayCan an observed event in fact be of zero probability?Unit random processmodels of coins and uniform distributionHow to get the number of successes given $n$ trials , probability $P$ and a random variable $X$Absorbing Markov chain in a computer. Is “almost every” turned into always convergence in computer executions?Stopped random walk is not uniformly integrable